LSAT and Law School Admissions Forum

Get expert LSAT preparation and law school admissions advice from PowerScore Test Preparation.

 Adam Tyson
PowerScore Staff
  • PowerScore Staff
  • Posts: 5191
  • Joined: Apr 14, 2011
|
#91127
In general, novthe13th, no, we do not suggest using the Negation Technique on Justify the Conclusion questions. The correct answer will be sufficient for the conclusion, but may not always be necessary for it, so negating it may not destroy the argument.

Here's an example:

"Adam has 10 children, so Adam has more children than Michael has."

To justify that conclusion, I could say "Michael has fewer than 10 children." Negating that WOULD ruin the argument: "Michael does not have fewer than 10 children" or "Michael has at least 10 children" would be negations of that answer, and they would ruin the argument. Certainly "Michael has fewer than 10 children" was a necessary assumption of the argument and also a sufficient one.

But another correct answer might be "Michael has no children." When we negate that we get "Michael has some children," and that does nothing to the argument. It's fine that Michael has children; Adam could still have more. So that wasn't a necessary assumption. But it's still a good answer for a Justify question, because if Michael has no children and Adam has 10, then Adam absolutely does have more children than Michael!

Don't use the Negation Technique on Justify questions. While it might work in some cases, it will lead you astray and make you reject correct answers in other cases. A test that only works some of the time is not a very good test!
User avatar
 ange.li6778
  • Posts: 34
  • Joined: Dec 27, 2021
|
#95036
I understand why A is the answer, but I feel like C also does what is required of this question. If some banks would lend to companies that were not financially strong five years ago, but does not lend at all to those companies now, wouldn't that also show that total lending now is less than 5 years ago? A is definitely a stronger answer that leaves no gap in the argument, so is that why A is right over C--because it's "more correct"?
 Robert Carroll
PowerScore Staff
  • PowerScore Staff
  • Posts: 1787
  • Joined: Dec 06, 2013
|
#95197
ange.li6778,

We already have in the stimulus that banks lent more to small and medium-sized companies five years ago. Answer choice (C) claims that banks would sometimes lend to companies that weren't financially strong...like those small and medium-sized companies? This is a Justify question, so the answer choice had better not allow my objection, which it does. Answer choice (C) is not necessarily positing any lending above and beyond what the stimulus already prove in its premises, so it's not necessarily true that answer choice (C) requires more net lending five years ago, as required to prove the conclusion.

Robert Carroll

Get the most out of your LSAT Prep Plus subscription.

Analyze and track your performance with our Testing and Analytics Package.